Recurrence relation for the Legendre functions

Click For Summary
The discussion focuses on deriving the recurrence relation for Legendre functions from Legendre's equation. The solution is assumed in the form of a power series, which is then substituted into the equation to yield a series of terms. The key point of confusion is how the transition from xn-2 to xn occurs when rearranging the terms, specifically involving the coefficient an+2. A detailed breakdown of the derivatives y' and y'' is provided to clarify how to collect terms and set coefficients to zero for each power of x. Understanding this process is crucial for finding solutions to Legendre's equation effectively.
aaaa202
Messages
1,144
Reaction score
2
My book wants to find solutions to Legendre's equation:

(1-x2)y'' - 2xy' 0 l(l+1)y = 0 (1)

By assuming a solution of the form:

y = Ʃanxn , the sum going from 0->∞ (2)

Now by plugging (2) into (1) one finds:

Ʃ[n(n-1)anxn-2-n(n-1)anxn - 2nanxn +
l(l+1)anxn = 0

Rearranging my book writes this as:

Ʃ[(n+2)(n+1)an+2 - (n(n+1)-l(l+1))an]xn = 0

I can't understand the emphasized bit. How did we go from xn-2 to xn using an+2?
 
Last edited:
Physics news on Phys.org
aaaa202 said:
My book wants to find solutions to Legendre's equation:

(1-x2)y'' - 2xy' 0 l(l+1)y = 0 (1)

By assuming a solution of the form:

y = Ʃanxn , the sum going from 0->∞ (2)

Now by plugging (2) into (1) one finds:

Ʃ[n(n-1)anxn-2-n(n-1)anxn - 2nanxn +
l(l+1)anxn = 0

Rearranging my book writes this as:

Ʃ[(n+2)(n+1)an+2 - (n(n+1)-l(l+1))an]xn = 0

I can't understand the emphasized bit. How did we go from xn-2 to xn using an+2?

Until you get the hang of these things the best policy is to write it out in a bit more detail: from y = a_0 + a_1 x + a_2 x^2 + \cdots + a_n x^n + \cdots, we have
y&#039; = a_1 + 2 a_2 x + 3a_3 x^2 + \cdots + (n+1) a_{n+1} x^n + \cdots, \\<br /> y&#039;&#039; = 2a_2 + 3\cdot 2 a_3 x + 4 \cdot 3 a_4 x^2 + \cdots + (n+2)(n+1) a_{n+2} x^n + \cdots. So now you can write out the terms of
(1-x^2) y&#039;&#039; - 2xy&#039; + l(l+1) y, collect all the terms in x^0, \: x^2, \: x^2, \ldots, x^n, \ldots and require them all to be zero.

RGV
 
Question: A clock's minute hand has length 4 and its hour hand has length 3. What is the distance between the tips at the moment when it is increasing most rapidly?(Putnam Exam Question) Answer: Making assumption that both the hands moves at constant angular velocities, the answer is ## \sqrt{7} .## But don't you think this assumption is somewhat doubtful and wrong?

Similar threads

  • · Replies 5 ·
Replies
5
Views
1K
  • · Replies 1 ·
Replies
1
Views
2K
Replies
1
Views
2K
Replies
8
Views
3K
Replies
5
Views
2K
Replies
3
Views
2K
  • · Replies 11 ·
Replies
11
Views
2K
  • · Replies 14 ·
Replies
14
Views
2K
  • · Replies 2 ·
Replies
2
Views
1K
  • · Replies 16 ·
Replies
16
Views
2K